Đến nội dung

Hình ảnh

Marathon số học Olympic

- - - - -

  • Please log in to reply
Chủ đề này có 191 trả lời

#1
Ego

Ego

    Thượng sĩ

  • Điều hành viên OLYMPIC
  • 296 Bài viết

Qua trao đổi với bạn No Moniker, Viet nam is in my heart và Bảo thì mình xin mở topic về số học này.
Mục đích của topic này là để trao đổi, trau dồi thêm về các bài toán số học ở cấp phổ thông, phục vụ cho việc thi HSG, Olympic,...

Sau đây là một số chủ đề có thể thảo luận trong topic này:

  • Các bài toán về chia hết
  • Phương trình nghiệm nguyên
  • Các bài toán liên quan đến hàm số học
  • Thặng dư chính phương - Ký hiệu Legendre, ký hiệu Jakobi
  • Cấp số nguyên - Căn nguyên thủy
  • Bất đẳng thức số học
  • Các bài toán số học liên quan đến tổ hợp
  • Bổ đề LTE
  • Các định lý số học như định lý Fermat, định lý Wilson, ...
  • Phần nguyên
  • Các bài toán liên quan đến định lý thặng dư Trung Hoa
  • ...

Nội dung của cuộc thi này khá đơn giản, khi bạn giải đúng được bài toán hiện có thì bạn có thể đăng lên tại đây và mình sẽ cộng thêm cho các bạn một điểm, và các bạn có quyền được đề xuất bài toán mới. Như vậy ai giải thì người đó sẽ có quyền đề xuất, trừ khi bạn không biết đề xuất bài nào thì bạn có thể nhờ hỗ trợ.

 

Và một số quy định yêu cầu các bạn tuân thủ:

  1. Chỉ cho phép các bài toán trong phạm vi số học
  2. Ghi nguồn bài toán rõ ràng
  3. Không được phép giải bài toán của chính mình đề xuất, không được phép đề xuất các bài toán trong các cuộc thi chưa kết thúc (ví dụ như tạp chí toán học & tuổi trẻ,...)
  4. Không được spam, lời giải rõ ràng, cụ thể.
  5. Khi bạn giải bài toán thứ $n$ thì bạn đề xuất luôn bài toán thứ $n + 1$ (đánh đúng số thứ tự). Sau đây là mẫu:
    Lời giải bài $n$. ABCXYZ
    Bài toán $n + 1$. (Nguồn) Cho ba số $a, b, c$. Chứng minh rằng $3\mid abc$.
  6. Lưu ý không đăng các bài toán mở, các giả thuyết, ...
  7. Nếu một bài toán trong vòng $7$ ngày chưa ai giải được thì sẽ được đánh dấu lại và mình sẽ đăng bài toán tiếp theo. Bất cứ lúc nào bạn muốn đề xuất lời giải cho bài chưa được giải cũng được và sẽ được cộng hai điểm nếu như lời giải đúng. Ngoài ra nếu các bạn nghĩ mình có lời giải hay hơn của bạn trước tiên giải bài nào đó thì xin cứ đăng (sẽ chỉ cộng điểm cho bạn làm đúng và nhanh nhất), như vậy sẽ học hỏi lẫn nhau được nhiều hơn.
    Ngoài ra, trước khi hết hạn $4$ ngày của một bài toán chưa được giải thì mong các bạn không đề xuất bài toán mới.
  8. Yêu cầu các bài toán có độ khó nhất định, phải suy nghĩ mới làm được.
  9. Yêu cầu tuân thủ các quy định. Bài viết nào có tính chất spam sẽ bị xóa đi hoặc lời giải đúng nhưng không rõ ràng, lan man sẽ chỉ nhận được $0,5$ điểm.

Mình khuyến khích mọi người tự đưa lời giải của chính mình thay vì lời giải của người khác hoặc dẫn link lời giải.

Hi vọng các bạn tham gia và đón nhận :D. Nếu các bài toán hay và lời giải đẹp thì ta sẽ tổng hợp thành một tài liệu nhỏ để tham khảo trong quá trình học Olympic, sẽ khá tốt.

Lưu ý: Các bạn khi đăng lời giải hãy để mọi người kiểm tra hộ bạn rồi hẳn đề xuất bài toán mới (kinh nghiệm của mình)


Bài viết đã được chỉnh sửa nội dung bởi Ego: 29-05-2016 - 21:57


#2
Ego

Ego

    Thượng sĩ

  • Điều hành viên OLYMPIC
  • 296 Bài viết

Bài toán hiện nay:
Bài toán 4*. (PEN I19) Cho $a,b,c,d$ là các số thực thỏa mãn $[na]+[nb]=[nc]+[nd]$ với mọi $n$ nguyên thế thì một trong ba số $a+b,a-c,a-d$ là số nguyên

Bài 50. (AMM) Chứng minh rằng nếu ta chọn $n$ số tự nhiên từ $2n$ số nguyên dương đầu tiên thì ta luôn tìm được số square - free.
Nói thêm về số square-free, ta gọi $n$ là số square-free nếu và chỉ nếu với $p\in\mathbb{P}$ sao cho $p\mid n$ thì $v_{p}(n) = 1$.

Điểm số hiện nay (cập nhật ngày 10/06/2016 post #158). 
 

22d287b57202779708490950a6be4fb5839eb40f

 

 


Bài viết đã được chỉnh sửa nội dung bởi Zaraki: 11-06-2016 - 16:27


#3
Ego

Ego

    Thượng sĩ

  • Điều hành viên OLYMPIC
  • 296 Bài viết

Bài toán 1. (Kazakhstan NMO 2010) Cho trước số tự nhiên $n$ sao cho tồn tại số tự nhiên $a$ thỏa mãn $a^{n - 1} \equiv 1\pmod{n}$ và với mọi ước nguyên tố $p$ của $n - 1$ thì $a^{\frac{n - 1}{p}} \not\equiv 1\pmod{n}$. Chứng minh rằng $n$ là số nguyên tố.



#4
Chris yang

Chris yang

    Thượng sĩ

  • Thành viên
  • 223 Bài viết

Bài toán 1. (Kazakhstan NMO 2010) Cho trước số tự nhiên $n$ sao cho tồn tại số tự nhiên $a$ thỏa mãn $a^{n - 1} \equiv 1\pmod{n}$ và với mọi ước nguyên tố $p$ của $n - 1$ thì $a^{\frac{n - 1}{p}} \not\equiv 1\pmod{n}$. Chứng minh rằng $n$ là số nguyên tố.

Gọi $x$ là $ord_n(a)$, suy ra $x|n-1$. Nếu $x$ là ước nhỏ hơn $n-1$ của $n-1$ thì hiển nhiên $a^x\not\equiv 1\pmod n$ ( theo giả thiết). Do đó $x=n-1$

Vì $n-1=ord_n(a)$ nên $n-1\leq \varphi (n)$. Mặt khác ta luôn có $n-1\geq \varphi (n)$ với mọi $n\in\mathbb{N}$ nên $n-1=\varphi (n)$, kéo theo $n$ là số nguyên tố.

Ta có đpcm

 

-----------------------------------------------------------------------------------------------------

Bài toán $2$. ( vở) : CMR với mọi $n\in\mathbb{N}^*$  tồn tại $k_0,k_1,....,k_n>1$ đôi một nguyên tố cùng nhau sao cho $k_0k_1....k_n-1$ là tích hai số nguyên liên tiếp.


Bài viết đã được chỉnh sửa nội dung bởi Ego: 26-05-2016 - 12:31


#5
Ego

Ego

    Thượng sĩ

  • Điều hành viên OLYMPIC
  • 296 Bài viết

Lời giải bài 2. Yêu cầu bài toán tương đương với việc chứng minh $a^{2} + a + 1$ luôn có ít nhất $n + 1$ ước nguyên tố, hay nói cách khác (khi đó ta có thể phân hoạch thành các số nguyên tố cùng nhau) cho $a \to +\infty$ thì tập các ước nguyên tố của $a^{2} + a + 1$ không bị chặn.
Dĩ nhiên $a^{2} + a + 1 \equiv 1\pmod{2}$, ta xét $A = (2a + 1)^{2} + 3$.
Theo định lý Dirichlete thì tồn tại vô hạn số nguyên tố $p$ có dạng $12k + 1$. Lúc đó $\left(\frac{-3}{p}\right) = 1$.
Gọi $n + 1$ số nguyên tố có dạng như vậy là $p_{0}, p_{1}, p_{2}, \cdots p_{n}$. (đây là các số lẻ)
Theo như trên thì với mỗi $p_{i}$ thì tồn tại $b_{i}$ sao cho $b_{i}^{2} + 3 \vdots p_{i}$ (*) và các số $x \equiv b_{i} \equiv p_{i}$ thì luôn thỏa (*). Nếu $b_{i}$ lẻ thì ta đặt $a_{i} = \frac{a_{i} - 1}{2}$, còn nếu $b_{i}$ chẵn thì ta xét $a_{i} = \frac{b_{i} + p_{i} - 1}{2}$. Lưu ý lúc này $(2a_{i} + 1)^{2} + 3 \vdots p_{i}$
Lúc này, áp dụng định lý thặng dư Trung Hoa ta luôn tìm được $a$ sao cho $a \equiv a_{i} \pmod{p_{i}}$.

Khi đó nếu $a^{2} + a + 1 = p_{0}^{m_{0}}p_{1}^{m_{1}}\cdots p_{n}^{m_{n}}.A$ với $A$ nguyên tố cùng nhau đôi một với $p_{i}$.
Lúc này ta chọn $k_{i} = p_{i}^{m_{i}}$ với mọi $0\le i \le n - 1$ và $k_{n} = Ap_{n}^{m_{n}}$

Bài toán 3. (AoPS) Giả sử $a, b$ là các số nguyên và $n$ là số tự nhiên thỏa $2^{n}a + b$ là số chính phương với mọi $n$. Chứng minh rằng $a = 0$

Nhắc nhở


Bài viết đã được chỉnh sửa nội dung bởi Ego: 24-05-2016 - 09:08


#6
hoanglong2k

hoanglong2k

    Trung úy

  • Điều hành viên THCS
  • 965 Bài viết

 Lời giải bài 3. Do $2^na+b$ là số chính phương với mọi $n$ nên $2^{n+2}a+4b$ và $2^{n+2}a+b$ cũng là số chính phương.

  Đặt $x^2=2^{n+2}a+4b$ và $y^2=2^{n+2}a+b$

  Nếu $a<0$, chọn $n$ đủ lớn để $2^na+b<0$ sẽ vô lí, nên $a\geq 0$, xét khi $a\neq 0$

  Với $b>0$, chọn $n$ sao cho $2^{n}> \dfrac{9b^2-10b+1}{16a}$, khi đó ta sẽ có 

  $4.2^{n+2}a>9b^2-10b+1\Leftrightarrow 4x^2>(3b+1)^2\Leftrightarrow 2x>3b+1\Leftrightarrow 2^{n+2}a+b>2^{n+2}a+4b-2x+1$

  $\Leftrightarrow y^2>(x-1)^2$

  Mặt khác thì $b>0$ nên $(x-y)(x+y)=3b>0$ nên $x>y$, hay $x^2>y^2>(x-1)^2$, vô lí

  Với $b<0$ thì $-3b-1>0$, chọn $n$ sao cho $2^{n}> \dfrac{9b^2-10b+1}{16a}$, khi đó ta sẽ có 

  $4.2^{n+2}a>9b^2-10b+1\Leftrightarrow 4x^2>(3b+1)^2\Leftrightarrow 2x>-3b-1\Leftrightarrow 2^{n+2}a+4b+2x+1>2^{n+2}a+b$

  $\Leftrightarrow (x+1)^2>y^2$

  Mặt khác $b<0$ nên $x<y$ hay $x^2<y^2<(x+1)^2$ vô lí

  Do đó $b=0$ hay $2^na$ là số chính phương với mọi $n$

  Cho $n=0$ và $n=1$ ta được $q^2=2a=2p^2$, điều này xảy ra khi và chỉ khi $p=q=0$ tức là $a=0$

 

 Chả biết nên đề xuất bài như thế nào :-s

 Bài 4. (AoPS - Bulgaria 1983) Chứng minh rằng nếu $a,b,c$ là các số thực thỏa mãn $[na]+[nb]=[nc]$ với mọi $n$ nguyên dương thì tồn tại ít nhất một trong ba số $a,b,c$ nguyên


Bài viết đã được chỉnh sửa nội dung bởi Ego: 24-05-2016 - 09:08


#7
I Love MC

I Love MC

    Đại úy

  • Thành viên nổi bật 2016
  • 1861 Bài viết

Bài 4 : Ta chứng minh $c=a+b$.
Giả sử $ c \ne a+b$. Xét hai trường hợp sau :
Trường hợp 1 : Xét $c>a+b$ khi đó $c-a-b>0$
Đặt $n_0=[\frac{1}{c-a-b}]+1$ suy ra $n_0 \in \mathbb{N^*}$ và $n_0>\frac{1}{c-a-b}$
Từ $c-a-b>0$ suy ra $n_0(c-a-b)>1$ nên $n_0c-1>n_0(a+b)$
Sử dụng tính chất về phần nguyên $[n_0c]>n_0c-1>n_0(a+b) \ge [n_0(a+b)] \ge [n_0a]+[n_0b]$
Điều này có nghĩa là tồn tại $n_0$ là số nguyên dương sao cho $[nc]>[na]+[nb]$ (mâu thuẫn)
Trường hợp 2 : Xét $c<a+b$ suy ra $a+b-c>0$
Đặt $n_0=[\frac{2}{a+b-c}]+1$ khi đó $n_0 \in \mathbb{N^*}$ và $n_0>\frac{2}{a+b-c}$
$a+b-c \Rightarrow n_0c<n_0(a+b)-2$
Theo tính chất phần nguyên : $[n_0c] \le n_0c<(n_0a-1)+(n_0b-1) \le [n_0a]+[n_0b]$
Lập luận như trên như vầy là tồn tại $n_0$ nguyên dương sao cho $[nc]<[na]+[nb]$ (mâu thuẫn)
Từ đó suy ra $c=a+b$
Như vậy ta có $[na]+[nb]=[na+nb]$
Mà theo một tính chất của phần nguyên thì $[x+y] \ge [x]+[y] \forall x,y \in \mathbb{R}$
Dấu bằng xảy ra khi $\left \{ x \right \}=\left \{ y \right \}=0$ (kí hiệu $\left \{ a \right \}$ là phần lẻ của $a$)
Từ đó suy ra $na,nb$ là số nguyên
Nếu $a,b$ không nguyên thì ta có $n(a+b)=nc$ là số nguyên suy ra $c$ là số nguyên. Còn nếu $a,b$ đều là số nguyên thì ta có đpcm.

 

 

Để chứng minh $c=a+b$ thì có lẽ đơn giản hơn nhiều! :)
Ta có: $[na]+[nb]=[nc]\Rightarrow n(a+b)-\left \{ na \right \}-\left \{ nb \right \}=nc-\left \{ nc \right \}\Rightarrow \left \{ nc \right \}-\left \{ na \right \}-\left \{ nb \right \}=n(c-a-b)$
Giả sử $c\neq a+b$. Do $1>\left \{ nc \right \}-\left \{ na \right \}-\left \{ nb \right \}>-2$ mà $a,b,c$ cố định nên chọn $n$ đủ lớn ta suy ra $\left | n(c-a-b) \right |>\left | \left \{ nc \right \}-\left \{ nb \right \}-\left \{ na \right \} \right |$ (mâu thuẫn) $\Rightarrow c=a+b$
Chỗ này chắc không ổn lắm, làm sao kết luận luôn $\left \{ x \right \}=\left \{ y \right \}=0$ được nhỉ? :mellow:
Theo mình đây mới là điểm khó của bài toán.

Mình cũng đang thiếu sự chặt chẽ trong lí luận . Cậu có thể sửa lời giải để có thêm chắc chắn


Bài viết đã được chỉnh sửa nội dung bởi Ego: 25-05-2016 - 12:04
$\LaTeX$


#8
baopbc

baopbc

    Himura Kenshin

  • Thành viên nổi bật 2016
  • 410 Bài viết

Để chứng minh $c=a+b$ thì có lẽ đơn giản hơn nhiều! :)

Ta có: $[na]+[nb]=[nc]\Rightarrow n(a+b)-\left \{ na \right \}-\left \{ nb \right \}=nc-\left \{ nc \right \}\Rightarrow \left \{ nc \right \}-\left \{ na \right \}-\left \{ nb \right \}=n(c-a-b)$

Giả sử $c\neq a+b$. Do $1>\left \{ nc \right \}-\left \{ na \right \}-\left \{ nb \right \}>-2$ mà $a,b,c$ cố định nên chọn $n$ đủ lớn ta suy ra $\left | n(c-a-b) \right |>\left | \left \{ nc \right \}-\left \{ nb \right \}-\left \{ na \right \} \right |$ (mâu thuẫn) $\Rightarrow c=a+b$

Từ đó suy ra $c=a+b$  
Như vậy ta có $[na]+[nb]=[na+nb]$ 
Mà theo một tính chất của phần nguyên thì $[x+y] \ge [x]+[y] \forall x,y \in \mathbb{R}$ 
Dấu bằng xảy ra khi ${x}={y}=0$ (kí hiệu ${a}$  là phần lẻ của $a$)  
Từ đó suy ra $na,nb$ là số nguyên 
Nếu $a,b$ không nguyên thì ta có $n(a+b)=nc$ là số nguyên suy ra $c$ là số nguyên. Còn nếu $a,b$ đều là số nguyên thì ta có đpcm 
 

Chỗ này chắc không ổn lắm, làm sao kết luận luôn $\left \{ x \right \}=\left \{ y \right \}=0$ được nhỉ? :mellow:

Theo mình đây mới là điểm khó của bài toán.



#9
bangbang1412

bangbang1412

    Độc cô cầu bại

  • Phó Quản lý Toán Cao cấp
  • 1667 Bài viết
Về phần chứng minh $c=a+b$ ta có tính chất phần nguyên nên $a+b+\frac{2}{n} \geq c \geq a+b-\frac{2}{n}$ lấy giới hạn hai vê có ngay $c=a+b$ .Bài toán trên là bài toán con của bài toán sau , cho $a,b,c,d$ là các số thực thỏa mãn $[na]+[nb]=[nc]+[nd]$ với mọi $n$ nguyên thế thì một trong ba số $a+b,a-c,a-d$ là số nguyên

$$[\Psi_f(\mathbb{1}_{X_{\eta}}) ] = \sum_{\varnothing \neq J} (-1)^{\left|J \right|-1} [\mathrm{M}_{X_{\sigma},c}^{\vee}(\widetilde{D}_J^{\circ} \times_k \mathbf{G}_{m,k}^{\left|J \right|-1})] \in K_0(\mathbf{SH}_{\mathfrak{M},ct}(X_{\sigma})).$$


#10
songuku

songuku

    Binh nhì

  • Thành viên mới
  • 10 Bài viết

Bài toán 4 là bài thi của Bulgari năm 1983

 

 

Để chứng minh $c=a+b$ thì có lẽ đơn giản hơn nhiều! :)
Ta có: $[na]+[nb]=[nc]\Rightarrow n(a+b)-\left \{ na \right \}-\left \{ nb \right \}=nc-\left \{ nc \right \}\Rightarrow \left \{ nc \right \}-\left \{ na \right \}-\left \{ nb \right \}=n(c-a-b)$
Giả sử $c\neq a+b$. Do $1>\left \{ nc \right \}-\left \{ na \right \}-\left \{ nb \right \}>-2$ mà $a,b,c$ cố định nên chọn $n$ đủ lớn ta suy ra $\left | n(c-a-b) \right |>\left | \left \{ nc \right \}-\left \{ nb \right \}-\left \{ na \right \} \right |$ (mâu thuẫn) $\Rightarrow c=a+b$
Chỗ này chắc không ổn lắm, làm sao kết luận luôn $\left \{ x \right \}=\left \{ y \right \}=0$ được nhỉ? :mellow:
Theo mình đây mới là điểm khó của bài toán.

Để xử lí việc này. ta có thể gải sử a,b thuộc [0,1). sau đó giả sử a và b khác 0, xét 2 trường hợp : trường hợp 1: là a và b hữu tỉ suy ra mâu thuẫn, trường hợp 2 : xét a hoặc b vô tỷ ( đây là việc khó nhất trong bài) cũng suy ra mâu thuẫn.

 

 

Vấn đề chính là nằm ở chỗ $a$ và $b$ cùng là số vô tỉ đó bạn, chỗ này mình không làm được

Không cần xét cả a và b vô tỉ, nếu cả a và b vô tỉ thì coi như chỉ có a vô tỉ. ok


Bài viết đã được chỉnh sửa nội dung bởi Ego: 25-05-2016 - 11:55


#11
hoanglong2k

hoanglong2k

    Trung úy

  • Điều hành viên THCS
  • 965 Bài viết

Để xử lí việc này. ta có thể gải sử a,b thuộc [0,1). sau đó giả sử a và b khác 0, xét 2 trường hợp : trường hợp 1: là a và b hữu tỉ suy ra mâu thuẫn, trường hợp 2 : xét a hoặc b vô tỷ ( đây là việc khó nhất trong bài) cũng suy ra mâu thuẫn.

  Vấn đề chính là nằm ở chỗ $a$ và $b$ cùng là số vô tỉ đó bạn, chỗ này mình không làm được 



#12
Ego

Ego

    Thượng sĩ

  • Điều hành viên OLYMPIC
  • 296 Bài viết

Lời giải bài 4. (mình vẫn có cảm giác không ổn, nhờ các bạn kiểm tra hộ) Như Bằng chứng minh phần trên đẩy về giới hạn kia là đẹp rồi. Mình xin phép được làm phần cuối như sau. Xét $\left\lfloor na\right\rfloor + \left\lfloor nb\right\rfloor = \left\lfloor n(a + b)\right\rfloor$ (*)
 

 

Bổ đề. Cho $a, b$ là hai số thực thuộc khoảng $(0; 1)$, khi đó tồn tại hai số nguyên dương $m, k$ để $$\left\lfloor\frac{m}{a}\right\rfloor = \left\lfloor\frac{k}{b}\right\rfloor = n$$

Chứng minh


Quay lại bài toán, để ý là $na + nb = n(a + b)$ nên ta đưa bài toán về thành $\{na\} + \{nb\} = \{n(a + b)\}$. Nhận xét là $\{n(a + b)\} < 1$. Tức là nếu ta chứng minh được mệnh đề sau: "Nếu $\{na\} + \{nb\} < 1$ đúng với mọi $n$ thì ít nhất một trong hai số $a, b$ nguyên" thì bài toán được giải quyết. Để chứng minh nó, cứ việc giả sử phản chứng, nghĩa là $a, b$ không nguyên.
Lại để ý thêm là ta có thể đưa $a, b$ về đoạn $[0; 1)$ nhờ tính chất $\{a + k\} = \{a\}$ nếu và chỉ nếu $k$ nguyên. Nhờ giả sử nên ta chỉ xét $a, b \in (0; 1)$:
 

  • Với $n = 1$ ta suy ra rằng $a + b < 1 \implies 1 - b > a$ (do $\{a\} = a$ và $\{b\} = b$). Chọn $\epsilon$ sao cho $1 > 1 - b > \epsilon > a > 0$. Từ đây ta có hai tính chất như sau $$\begin{cases}\frac{1 - \epsilon}{b} > 1 \\ \frac{\epsilon}{a} > 1\end{cases}$$
  • Theo bổ đề thì ta chọn $n$ như trên. Lúc này $n < \frac{m}{a}$, mặt khác do $\frac{\epsilon}{a} > 1$ nên $\frac{m - \epsilon}{a} < n$. Tóm lại $\frac{m - \epsilon}{a} < n < \frac{m}{a} \iff m - \epsilon < na < m$ (lưu ý do $\epsilon \in (0, 1)$ nên từ đây suy ra $\{na\} > 1 - \epsilon$), tương tự ta cũng có $\frac{k - (1 - \epsilon)}{b} < n < \frac{k}{b} \implies k - (1 - \epsilon) < bn < k \implies \{bn\} > \epsilon$
  • Cộng hai vế lại tương ứng thu được $\{an\} + \{bn\} > (1 - \epsilon) + \epsilon = 1$, điều này sẽ vô lí. Tóm lại có ít nhất $a, b$ là số nguyên.

Bài toán 5. (Mock Canada 2008) Với số nguyên $p$ và số nguyên dương $n$, quy ước $v_{p}(n)$ là lũy thừa đúng của $p$ trong khai triển chính tắc của $n$. Cho số nguyên dương $d$ và tập hữu hạn $\{p_{1}, p_{2}, \cdots , p_{k}\}$ các số nguyên tố. Chứng minh rằng có vô hạn số nguyên dương $n$ sao cho $d\mid v_{p_{i}}(n!)$ đúng với mọi $1 \le i \le k$.


Bài viết đã được chỉnh sửa nội dung bởi bangbang1412: 07-06-2016 - 14:20


#13
bangbang1412

bangbang1412

    Độc cô cầu bại

  • Phó Quản lý Toán Cao cấp
  • 1667 Bài viết
Mong Ego giải nốt bài $4$ số mình đề cập trên , còn về bổ đề của Ego là hệ quả của định lý xấp xỉ Dirichle .( cho bạn nào tìm hiểu thêm )

$$[\Psi_f(\mathbb{1}_{X_{\eta}}) ] = \sum_{\varnothing \neq J} (-1)^{\left|J \right|-1} [\mathrm{M}_{X_{\sigma},c}^{\vee}(\widetilde{D}_J^{\circ} \times_k \mathbf{G}_{m,k}^{\left|J \right|-1})] \in K_0(\mathbf{SH}_{\mathfrak{M},ct}(X_{\sigma})).$$


#14
Visitor

Visitor

    Hạ sĩ

  • Thành viên
  • 66 Bài viết

Lời giải bài 5:

Ta sử dụng bổ đề quen thuộc : $v_{p}((qp^k+r)!)=v_{p}(qp^k)+v_{p}(r)$ với $r<p^k$, $p$ là số nguyên tố, $q,r$ là số tự nhiên.

C/m: $v_{p}((qp^k+r)!)=\frac{qp^k+r-S_{p}(qp^k+r)}{p-1}$

Do $r<p^k$ nên $r$ ko thể có biểu diễn quá $k$ chữ số trong hệ cơ số $p$ nên $S_{p}(qp^k+r)= S_{p}(qp^k)+S_{p}(r)$

suy ra: $v_{p}((qp^k+r)!)=\frac{qp^k+r-S_{p}(qp^k)-S_{p}(r)}{p-1}= \frac{qp^k-S_{p}(qp^k)}{p-1}+\frac{r-S_{p}(r)}{p-1}=v_{p}((qp^k)!)-v_{p}(r!)$ (đpcm)

Trở lại bài toán: 

Xét khai triển một số $n$ bất kì $n=p_{1}^{a_1}p_{2}^{a_2}...p_{k}^{a_k}.Â$

Theo $mod d$ thì ta thấy sẽ có hữu hạn bộ số $(a_1,a_2,...a_k)$ , cụ thể là $d^k$ bộ. Vì thế khi $n$ tăng đến vô cùng thì sẽ có vô số bộ bị lặp lại theo $modd$.

Xét các số dạng $a_s=(p_1p_2...p_k)^s$, theo nhận xét trên thì sẽ tồn tại các số $n_1<n_2<...<n_d$ sao cho : 

$v_{p_t}((a_{n_i})!)\equiv v_{p_t}((a_{n_j})!)(mod.d)$ với mọi $t$ từ $1$ đến $k$ và mọi $1\leq i,j\leq d$.

Vì các $n_i$ có thể lớn tùy ý nên ta chọn sao cho các điều kiện của bổ đề được thỏa mãn:

$a_{n_1}<min(p_{t}^{n_2})$

$a_{n_1}+a_{n_2}<min(p_{t}^{n_3})$

$a_{n_1}+a_{n_2}+a_{n_3}<min(p_{t}^{n_4})$

$........$

Áp dụng bổ đề ta có với mọi $p_t$ thì: 

$v_{p_t}((a_{n_2}+a_{n_1})!)=v_{p_t}((a_{n_2})!)+v_{p_t}((a_{n_1})!) \equiv 2v_{p_t}((a_{n_1})!)   (mod .p_t)$

$v_{p_t}((a_{n_3}+a_{n_2}+a_{n_1})!)=v_{p_t}((a_{n_3})!)+v_{p_t}((a_{n_2}+a_{n_1})!)\equiv 3v_{p_t}((a_{n_1})!) $

$........$

$v_{p_t}((a_{n_d}+...+a_{n_2}+a_{n_1})!)\equiv dv_{p_t}((a_{n_1})!) \equiv0$

CHọn $n=a_{n_d}+...+a_{n_2}+a_{n_1}$ thì $n$ thỏa đề. 
Vì $n_i$ lớn tùy ý nên cũng sẽ có vô số $n$ thỏa mãn đề bài.đpcm.

#

CHo mình nợ bài 6 vài phút.

Chả biết chọn bài nào, chọn đại 1 bài vui vui vậy :)

 

 

Bài toán 6. ( Toán cổ) Tìm tất cả số tự nhiên $n$ để có thể phân hoạch tập $S=\{1,2,3...,4n\}$ thành các tập con $4$ phần tử sao cho trong mỗi tập tồn tại một phần tử bằng trung bình cộng của ba phần tử còn lại.


Bài viết đã được chỉnh sửa nội dung bởi Visitor: 26-05-2016 - 13:59

__________

Bruno Mars


#15
bangbang1412

bangbang1412

    Độc cô cầu bại

  • Phó Quản lý Toán Cao cấp
  • 1667 Bài viết

Sau đây là một số điều có thể các bạn muốn đọc về bài toán này , ngoài lời giải của Ego thì vẫn còn một số lời giải khác và bài toán tổng quát , nhưng lời giải và phần cuối topic đó có vẻ không rõ ràng bạn nào xử nốt được thì hay .

http://www.artofprob...6h150720p849765

http://www.artofprob...nity/c6h1178349

http://artofproblems...h276500p1497267

Còn về bài toán tổng quát , ban đầu mình phát hiện nó ở tài liệu này rất hay các bạn có thể giải  :D  :D  nhưng đừng có đăng vào topic này

https://www.math.mun...en-20070711.pdf

Rất mong có một lời giải thích phù hợp cho lý luận cuối ở topic này để bài toán trọn vẹn 

http://www.artofprob...6h150720p849765


Bài viết đã được chỉnh sửa nội dung bởi bangbang1412: 26-05-2016 - 13:24

$$[\Psi_f(\mathbb{1}_{X_{\eta}}) ] = \sum_{\varnothing \neq J} (-1)^{\left|J \right|-1} [\mathrm{M}_{X_{\sigma},c}^{\vee}(\widetilde{D}_J^{\circ} \times_k \mathbf{G}_{m,k}^{\left|J \right|-1})] \in K_0(\mathbf{SH}_{\mathfrak{M},ct}(X_{\sigma})).$$


#16
Zaraki

Zaraki

    PQT

  • Phó Quản lý Toán Cao cấp
  • 4273 Bài viết

Bài toán 6. ( Toán cổ) Tìm tất cả số tự nhiên $n$ để có thể phân hoạch tập $S=\{1,2,3...,4n\}$ thành các tập con $4$ phần tử sao cho trong mỗi tập tồn tại một phần tử bằng trung bình cộng của ba phần tử còn lại.

Lời giải. Ta chứng minh rằng tập tất cả các số chẵn là tập các số $n$ cần tìm. 

 

Tập $S$ phân hoạch thành các tập con $A_i= \left \{ a_i,b_i,c_i,d_i \right \}$ với $3d_i=a_i+b_i+c_i$. Khi đó $$2n(4n+1)=\sum_{i=1}^{4n}i= \sum_{j=1}^n(a_j+b_j+c_j+d_j)= 4 \sum_{i=1}^n d_i.$$

Khi đó ta suy ra $2 \mid n$.

Bây giờ, ta sẽ chứng minh tồn tại một cách phân hoạch thoả mãn cho mọi $2 \mid n$ bằng quy nạp.Với $n=2$ thì ta chia thành hai tập $\{ 1,3,4,8 \}$ ($1+3+8=3 \cdot 4$) và $\{ 2,5,6,7 \}$ ($2+6+7=3 \cdot 5$). Nếu bài toán đúng với $n=2k$, tức tồn tại $2k$ tập $A_1,A_2, \cdots , A_{2k}$ thoả mãn bài toán. Do đó với $n=2k+2$, ta có thể phân hoạch $S$ thành $2k+2$ tập con $A_1,A_2, \cdots, A_{2k}, \{ 8k+1,8k+3,8k+4,8k+8 \} , \{ 8k+2,8k+5,8k+6,8k+7 \}$.

Vậy với mọi $n$ chẵn thì bài toán thoả mãn.

 

Bài toán 7. [1989 Saint Petersburg MO] Cho $n$ là số nguyên dương. Chứng minh rằng tất cả các số lớn hơn $\tfrac{n^4}{16}$ có thể viết thành tối đa một cách dưới tích của hai ước nguyên dương của số đó mà có hiệu không vượt quá $n$.


Discovery is a child’s privilege. I mean the small child, the child who is not afraid to be wrong, to look silly, to not be serious, and to act differently from everyone else. He is also not afraid that the things he is interested in are in bad taste or turn out to be different from his expectations, from what they should be, or rather he is not afraid of what they actually are. He ignores the silent and flawless consensus that is part of the air we breathe – the consensus of all the people who are, or are reputed to be, reasonable.

 

Grothendieck, Récoltes et Semailles (“Crops and Seeds”). 


#17
I Love MC

I Love MC

    Đại úy

  • Thành viên nổi bật 2016
  • 1861 Bài viết
Bài toán 7. [1989 Saint Petersburg MO] Cho $n$ là số nguyên dương. Chứng minh rằng tất cả các số lớn hơn $\tfrac{n^4}{16}$ có thể viết thành tối đa một cách dưới tích của hai ước nguyên dương của số đó mà có hiệu không vượt quá $n$. 

 

Lời giải bài 7.
Giả sử rằng $ab=cd>\frac{n^4}{16}$ với $a>c,d$ và $n \ge a-b$ 
Đặt $p=(a,c)$ nên vậy $a=pq,c=pr$ ta có thể  tìm $p,q,r,s$ sao cho : 
$a=pq,c=pr,b=rs,d=qs$. 
Ta có $a>c$ suy ra $q>r$ tương tự $a>d \Rightarrow p>s$ 
Ta có $n \ge a-b=pq-rs \ge (r+1)(s+1)-rs=r+s+1 \ge 2\sqrt{b}+1$ (bất đẳng thức Cosi) 
Vậy nên $(\frac{n-1}{2})^2 \ge b$ và $a \le n+b \le (\frac{n-1}{2})^2+n=(\frac{n+1}{2})^2$ 
Suy ra $ab \le (\frac{n^2-1}{4})^2 <\frac{n^4}{16}$ (mâu thuẫn) 
Bài toán 8 : (Mathematical Excalibur of Hong Kong)  
Giải phương trình nghiệm nguyên dương : 
$x^2y^2=z^2(z^2-x^2-y^2)$

 


Bài viết đã được chỉnh sửa nội dung bởi I Love MC: 26-05-2016 - 17:02


#18
Long Phi

Long Phi

    Binh nhất

  • Thành viên
  • 26 Bài viết

http://diendantoanho...x2y2z2x2-x2-y2/

Nhìn quen quen, lục lại tài liệu cũ thấy 

 

PQT: Mình không nghĩ bài này vi phạm nội quy, ít nhất cũng để cái link để người ta biết bài 8 có người giải.


Bài viết đã được chỉnh sửa nội dung bởi Zaraki: 26-05-2016 - 18:06


#19
bangbang1412

bangbang1412

    Độc cô cầu bại

  • Phó Quản lý Toán Cao cấp
  • 1667 Bài viết

Hai lời giải cho bài $3$ và bài $7$

Lời giải bài $3$ 

Nếu $a$ khác $0$ và $b=0$ , thì một trong hai sô $2a+b$ và $4a+b$ là số chính phương hay $a=2t^{2}$ nên $4a^{2}=8t^{2}$ là scp khi và chỉ khi $a=0$ 

Nếu $a,b$ khác $0$ xét dãy $(x_{n},y_{n})=(2\sqrt{2^{n}a+b},\sqrt{2^{n+2}a+b})$ thỏa $(x_{n}+y_{n}(x_{n}-y_{n})=3b$ , để ý rằng $x_{n},y_{n}$ đều nguyen nên $x_{n}+y_{n}|3b$ , cái này không xảy ra nếu $a$ khác $0$ vì khi đó dãy kia tăng vô hạn do đó $a=0$

Lời giải bài $7$

Giả sử tồn tại $a,b,c,d$ thỏa $a>c\geq d>b$ thỏa mãn $a-b\leq n$ và $ab=cd > \frac{n^{4}}{16}$ .Đặt $p=a+b,q=a-b,r=c+d,s=c-d$

Ta có $p^{2}-q^{2}=4ab=4cd=r^{2}-s^{2}>\frac{n^{4}}{4}$

Lại có $p^{2}-r^{2}=q^{2}-s^{2} \leq q^{2}\leq n^{2}$ , mà $r^{2}>r^{2}-s^{2}>\frac{n^{4}}{4}$ nên $r > \frac{n^{2}}{2}$ , lại có $p>r$ vì $q>s$

Nên $p^{2}-r^{2}>(\frac{n^{2}}{2}+1)^{2}-\frac{n^{4}}{4}\geq n^{2}+1$ vô lý nên có đpcm.
 

Bài toán 9:

Với mỗi số thực $\alpha$ khác $1$ thì ta định nghĩa tập $S(\alpha) = ([na] | n \in  Z)$ . Chứng minh tập $N$ không chia thành hợp của ba tập rời nhau dạng trên được.


Bài viết đã được chỉnh sửa nội dung bởi Ego: 31-05-2016 - 11:17

$$[\Psi_f(\mathbb{1}_{X_{\eta}}) ] = \sum_{\varnothing \neq J} (-1)^{\left|J \right|-1} [\mathrm{M}_{X_{\sigma},c}^{\vee}(\widetilde{D}_J^{\circ} \times_k \mathbf{G}_{m,k}^{\left|J \right|-1})] \in K_0(\mathbf{SH}_{\mathfrak{M},ct}(X_{\sigma})).$$


#20
bangbang1412

bangbang1412

    Độc cô cầu bại

  • Phó Quản lý Toán Cao cấp
  • 1667 Bài viết
EGO : Sửa thành số khác $1$ nhé cậu

Bài viết đã được chỉnh sửa nội dung bởi bangbang1412: 26-05-2016 - 18:37

$$[\Psi_f(\mathbb{1}_{X_{\eta}}) ] = \sum_{\varnothing \neq J} (-1)^{\left|J \right|-1} [\mathrm{M}_{X_{\sigma},c}^{\vee}(\widetilde{D}_J^{\circ} \times_k \mathbf{G}_{m,k}^{\left|J \right|-1})] \in K_0(\mathbf{SH}_{\mathfrak{M},ct}(X_{\sigma})).$$





0 người đang xem chủ đề

0 thành viên, 0 khách, 0 thành viên ẩn danh